Willkommen, Gast ( Anmelden | Registrierung )

Zurück zum Board Index
2 Seiten V   1 2 >  
Reply to this topicStart new topic
> Ab welcher Entfernung fliegen Geschosse senkrecht?, Gibt es da Tabellen für?
GFF_phoenix
Beitrag 3. Jan 2005, 16:44 | Beitrag #1
+Quote PostProfile CardPM
Major
Beiträge: 6.891



Gruppe: Members
Mitglied seit: 12.03.2003


gehen wir davon aus, wir stehen auf einem unbegrenzt hohen turm. oben ist ein abschussgerät (keine wirkliche waffe, hat aber die für das kaliber am besten geeignete lauflänge usw., son ding wird auch  für beschusstests benutzt)
wenn wir jetzt genau im winkel von 90° schießen, ab welcher entfernung fällt das geschoss senkrecht mit 180° vom himmel? also keinerlei vortrieb mehr, sondern es spielt nur noch die erdanziehungskraft eine rolle.

sagen wir als geschosse:
9x19mm
7,62x51mm
usw kaliber sind mir im großen und ganzen egal. interessiert mich einfach.

formeln sind nett, nur dann genau erklärt was was ist usw. durch balistische formeln steige ich nicht wirklich durch  :(


--------------------
Ludimus non laedimus!

 
jever
Beitrag 3. Jan 2005, 17:07 | Beitrag #2
+Quote PostProfile CardPM
Oberleutnant
Beiträge: 1.552



Gruppe: Members
Mitglied seit: 31.01.2003


Hat nix mit Balistik zu tun, sondern nur mit Reibung. Senkrecht nach unten fliegt das Geschoss, wenn der komplette Vortrieb aus dem Schuss verbraucht ist, dann wirkt nurnoch der Vektor nach unten, vorher wirken sie gemeinsam.
Die Formel ist ein Integral, das ich erstmal herleiten muss. Einfacher wäre es, wenn Du die maximale theoretische Schussweite hättest. Das wäre dann identisch. Da braucht man nämlich einige Werte, die ich net so ohne Weiteres aus dem Inet finden kann.

MfG jever
 
Adrehn
Beitrag 3. Jan 2005, 17:13 | Beitrag #3
+Quote PostProfile CardPM
Oberleutnant
Beiträge: 2.408



Gruppe: VIP
Mitglied seit: 08.09.2001


QUOTE(jever @ 03.01.2005, 17:07)
... nur mit Reibung. ....

... und Erdanziehungskraft* - schliesslich würden die Geschosse auch ohne Lufthülle irgendwann wieder runterkommen.

Den Rest überlass ich den Physikern

* Geschwindigkeit sinkt um 9,81 m/s pro Sekunde - und sobald sie negativ wird - hmm...


--------------------
Keine halben Sachen!
Wenn schon kein Gutmensch dann mit Konsequenz!
 
Ta152
Beitrag 3. Jan 2005, 17:17 | Beitrag #4
+Quote PostProfile CardPM
Oberst
Beiträge: 14.979



Gruppe: WHQ
Mitglied seit: 22.02.2002


@Jever: Bin mir ziemlich sicher das die weite nicht mit der (normalen) maximalen Schußweite übereinstimmen wird.


--------------------
/EOF
 
Herzog Atreides
Beitrag 3. Jan 2005, 17:19 | Beitrag #5
+Quote PostProfile CardPM
Hauptmann
Beiträge: 3.381



Gruppe: VIP
Mitglied seit: 10.12.2001


Nein, die Erdanziehungskraft spielt hier keine Rolle, da er in seiner Aufgabe von einem idealisiert unendlich hohen Turm ausgeht. Die Erdanziehung wirkt immer nur nach unten. UNd ist somit für die Vorwärstkomponente unerheblich. Die Enternung hängt also wirklich nur von der Reibung ab. Die Gleichung hierfür muß jetzt einer der Aerodynamiker rausholen, davon habe ich absolut keine Ahnung.

EDIT: Achso das genze Experiment nennt man dann einen waagrechten Wurf.


--------------------
  • Personalführung ist die Kunst, den Mitarbeiter so schnell über den Tisch zu ziehen, daß er die Reibungshitze als Nestwärme empfindet.
  • Leben in der Lage
  • Im Kampf sollt ihr beides sein, Feuer und Eis
 
jever
Beitrag 3. Jan 2005, 17:21 | Beitrag #6
+Quote PostProfile CardPM
Oberleutnant
Beiträge: 1.552



Gruppe: Members
Mitglied seit: 31.01.2003


@Adrehn: Die Frage kann nur über die Reibung beantwortet werden, da ich ohne diese eine Parabel habe, und keine senkrechte Gerade. Die Geschosse würden in einem Winkel =/= 180° aufkommen, und das ist ja die Ausgangsfrage.Die Erdanziehungskraft ist nur eine auslenkende Kraft, welche auf die Berechnung dess Erreichens des senkrechten Falles keinen direkten Einfluss hat.

MfG jever
 
jever
Beitrag 3. Jan 2005, 17:29 | Beitrag #7
+Quote PostProfile CardPM
Oberleutnant
Beiträge: 1.552



Gruppe: Members
Mitglied seit: 31.01.2003


Wobei, jetzt spielen wir mal Physiker und arbeiten mit Näherung, wenn Dir das genügt, Phönix:
Die Wurfweite beim horizontalen Wurf wird mit Wurfweite = Anfangsgeschwindigkeit x Wurzel 2 x Höhe / Erdanziehungskraft g berechnet.
Wenn wir die Höhe hoch genug nehmen, ändert sich die Wurfweite nurnoch minimal.
Nimm für die Höhe in Etwa das 5-10fache der maximalen Schussreichweite (je nach Grundlage), und Du hast eine Näherung.

MfG jever

edit: Erstmal sorry fürs Doppelpost,
dann: Hab grad mal in ein Datenblatt geschaut, DP MG 28 "Degtjarew": Grösste Schussweite 3500m, die nimmst Du mal 5 als Höhe. Damit kommst Du in etwa hin, sofern Du es nicht genau brauchst. Ein wenig probieren, z.B. nochmal das sechsfache verwenden, und schauen, ob die Wurfweiten noch wirklich unterschiedlich sind.
 
GFF_phoenix
Beitrag 3. Jan 2005, 17:32 | Beitrag #8
+Quote PostProfile CardPM
Major
Beiträge: 6.891



Gruppe: Members
Mitglied seit: 12.03.2003


ne, eigentlich will ich es genau wissen. so wie oben beschrieben. allerdings verstehe ich nicht, wenn das geschoss keinerlei waagerechten vortrieb hat, also waagerecht gesehen steht. dann wirkt doch die erdanziehungskraft, das es runterfeld. und genau in den moment wo es beginnt runterzufallen ohne jeglichen waagerechten vortrieb, möchte ich wissen, wie weit es geflogen ist.


--------------------
Ludimus non laedimus!

 
jever
Beitrag 3. Jan 2005, 17:38 | Beitrag #9
+Quote PostProfile CardPM
Oberleutnant
Beiträge: 1.552



Gruppe: Members
Mitglied seit: 31.01.2003


Was Du wissen willst ist, wann die Mündungsenergie von der Luftreibung aufgebraucht wurde.
Einmal kannst Du das durch meinen Vorschlag per Näherung bestimmen, oder aber eben ich such mal die Formel raus.
Die Erdanziehungskraft wirkt permanent, die Kugel beschreibt ja eine Parabel. Und wenn Du nur den Vortrieb haben willst, dann ist die Erdanziehung irrelevant, Du willst ja nur wissen,wann die Kugel ihren Vortrieb komplett in Reibung umgewandelt hat.

MfG jever
 
GFF_phoenix
Beitrag 3. Jan 2005, 17:45 | Beitrag #10
+Quote PostProfile CardPM
Major
Beiträge: 6.891



Gruppe: Members
Mitglied seit: 12.03.2003


so kann man es auch sagen smile.gif


--------------------
Ludimus non laedimus!

 
Adrehn
Beitrag 3. Jan 2005, 18:01 | Beitrag #11
+Quote PostProfile CardPM
Oberleutnant
Beiträge: 2.408



Gruppe: VIP
Mitglied seit: 08.09.2001


Aso ich dachte er will senkrecht nach oben schiessen - mein Fehler.


--------------------
Keine halben Sachen!
Wenn schon kein Gutmensch dann mit Konsequenz!
 
GFF_phoenix
Beitrag 3. Jan 2005, 18:06 | Beitrag #12
+Quote PostProfile CardPM
Major
Beiträge: 6.891



Gruppe: Members
Mitglied seit: 12.03.2003


nein, habe ich auch im anfangspost erwähnt. 90° waagerecht. wink.gif


--------------------
Ludimus non laedimus!

 
Adrehn
Beitrag 3. Jan 2005, 18:09 | Beitrag #13
+Quote PostProfile CardPM
Oberleutnant
Beiträge: 2.408



Gruppe: VIP
Mitglied seit: 08.09.2001


QUOTE(GFF_phoenix @ 03.01.2005, 16:44)
...
wenn wir jetzt genau im winkel von 90° schießen,
...

Also wenn Du s ganz genau haben willst...

Wenn ich eine Artelleriekanone mit 90° schiessen lasse, dann ist das senkrecht nach oben. Und von waagerecht stand da nix....

A'ight?


--------------------
Keine halben Sachen!
Wenn schon kein Gutmensch dann mit Konsequenz!
 
GFF_phoenix
Beitrag 3. Jan 2005, 18:15 | Beitrag #14
+Quote PostProfile CardPM
Major
Beiträge: 6.891



Gruppe: Members
Mitglied seit: 12.03.2003


hmm, ich habe mich dann falsch orientiert. hast aber recht. ich dachte an nen kompass und nicht an nen höhenmesser. wallbash.gif

also 0° wink.gif


--------------------
Ludimus non laedimus!

 
jever
Beitrag 3. Jan 2005, 18:26 | Beitrag #15
+Quote PostProfile CardPM
Oberleutnant
Beiträge: 1.552



Gruppe: Members
Mitglied seit: 31.01.2003


F=maµ=1/2p(cW)Av²
m=Masse, a= Beschleunigung, µ= Gleitreibungszahl, p= Luftdichte, cW= Luftwiderstandsbeiwert, A Frontfläche, v= Geschwindigkeit, s= Weg
Aus F=maµ wird v= Wurzel 2saµ oder s = v²/2aµ
So in Etwa sieht das Ganze aus. Willst Du wirklich, dass ich das Alles erstmal umforme? Hast Du µ und cW-Wert bzw. kannst die auftreiben?

MfG jever
 
GFF_phoenix
Beitrag 3. Jan 2005, 18:31 | Beitrag #16
+Quote PostProfile CardPM
Major
Beiträge: 6.891



Gruppe: Members
Mitglied seit: 12.03.2003


ich hab von sowas null plan. ich kann alles mögliche, ausser so komplizierte formeln behersschen. irgendwann macht es klack und ich raff das, aber bis da ist es mehr wie haarig. wink.gif

nein, ich hab dazu keine werte. kreuz as oder father christmas aber bestimmt. die kennen sich doch mit sowas aus.


--------------------
Ludimus non laedimus!

 
jever
Beitrag 3. Jan 2005, 18:37 | Beitrag #17
+Quote PostProfile CardPM
Oberleutnant
Beiträge: 1.552



Gruppe: Members
Mitglied seit: 31.01.2003


Naja, das Prob ist: Sone Berechnung ist halt nicht leicht zu finden, und für die Formelumstellung brauch ich ca. eine Stunde, sollte ich nicht tatsächlich was finden.
Was sich mir dabei aufdrängt ist die Frage, warum Du sowas auf den Meter genau wissen willst. Meine Vereinfachung genüft Ansprüchen bis ca. 100-200 Meter Genauigkeit. Also raus mit der Sprache tounge.gif

MfG jever
 
Radar O'Reilly
Beitrag 3. Jan 2005, 18:56 | Beitrag #18
+Quote PostProfile CardPM
Leutnant
Beiträge: 963



Gruppe: Members
Mitglied seit: 13.11.2003


Wieso lädst du dir nicht einfach einen kostenlosen Ballistikrechner runter?


--------------------
Bienenkotze schmeckt wie Honig
 
dr.ignaz
Beitrag 3. Jan 2005, 19:11 | Beitrag #19
+Quote PostProfile CardPM
Fähnrich
Beiträge: 123



Gruppe: Members
Mitglied seit: 07.06.2003


QUOTE(GFF_phoenix @ 03.01.2005, 17:32)
[..., eigentlich will ich es genau wissen... ]

für diese versuchsanordnung kannst du keinerlei realistische werte ermitteln.

als zweiten lösungsansatz kann man auch errechnen wann die kinetische nergie vollständig in wärmeenergie umgewandelt wurde.

wobei diese berechnung insgesamt zu einem der wirklichkeit-fremden ergebnis führt, da man eine umgebung wählt die nicht der erdumgebung entsprechen.

aus diesem grund lohnt sich das bestimmen der reichweite nicht. oder jemand hat einfach viel zuviel zeit oder hat lust das in einer physik I vorlesung mal durchzurechnen.

gruß

iggy
 
Yoda
Beitrag 3. Jan 2005, 22:47 | Beitrag #20
+Quote PostProfile CardPM
Leutnant
Beiträge: 904



Gruppe: Members
Mitglied seit: 04.03.2003


QUOTE
Hast Du µ und cW-Wert bzw. kannst die auftreiben?

Leider ist zumindest der cW-Wert (für µ weiss ichs nicht genau) geschwindigkeitsabhängig, damit kommt man mit der einfachen Formel für die Luftwiderstandskraft nicht weiter. Müsste man also entweder numerisch lösen oder wenn cW(v) als Formel bekannt ist, analytisch. Wird aber dann vermutlich irgendein fieses Integral.
 
dr.ignaz
Beitrag 3. Jan 2005, 23:16 | Beitrag #21
+Quote PostProfile CardPM
Fähnrich
Beiträge: 123



Gruppe: Members
Mitglied seit: 07.06.2003


werden bei der formel auch schon die bremsenden verwirbelungen hinter dem projektl berücksichtigt ? =)
zudem ist der wiederstandswert abhängig von der höhe des unendlichhohen turms.
nennt mich spaßbremse - aber es macht wenig sinn einen genauen wert für ein weltfremdes szenario zu ermitteln =) außer jemand hat spaß und zeit =)
 
GFF_phoenix
Beitrag 3. Jan 2005, 23:33 | Beitrag #22
+Quote PostProfile CardPM
Major
Beiträge: 6.891



Gruppe: Members
Mitglied seit: 12.03.2003


ich kann mir nicht vorstellen, das das so kompliziert sein soll. mata.gif


--------------------
Ludimus non laedimus!

 
dr.ignaz
Beitrag 3. Jan 2005, 23:57 | Beitrag #23
+Quote PostProfile CardPM
Fähnrich
Beiträge: 123



Gruppe: Members
Mitglied seit: 07.06.2003


die berechnung ansichist nicht schwer - vorraussetzung dafür ist, dass du den benauen strümungswiederstand des betreffenden projektils, dann noch die dichte des fluids/gases und noch ein paar kleinigkeiten.
wenn mann das kann, kann man sich am besten für den studiengang luft und raumfahrttechnik einschreiben und wird höchstwarscheinlich damit sehr leicht zurechtkommen.

die frage ist nur, was du mit einer solchen berechnung willst ? =)
 
GFF_phoenix
Beitrag 4. Jan 2005, 00:00 | Beitrag #24
+Quote PostProfile CardPM
Major
Beiträge: 6.891



Gruppe: Members
Mitglied seit: 12.03.2003


ich will das einfach wissen, wie weit fliegen die geschosse unter laborbedingungn wenn ich sie waagerecht abschieße, bis sie nur noch runterfallen ohne vortrieb. wink.gif


--------------------
Ludimus non laedimus!

 
Yoda
Beitrag 4. Jan 2005, 00:07 | Beitrag #25
+Quote PostProfile CardPM
Leutnant
Beiträge: 904



Gruppe: Members
Mitglied seit: 04.03.2003


QUOTE
ich kann mir nicht vorstellen, das das so kompliziert sein soll.

Ist es aber. Wenn du das als reines geometrisches Problem siehst (Wurfparabel) ist es simpel. Allerdings wird die Bahn dann nie wirklich senkrecht. Wenn du aber ein realistisches Ergebnis haben willst, spielt die Luftreibung da rein (die von der Geschwindigkeit, der genauen Projektilform, der Oberflächenbeschaffenheit des Projektils, der Luftdichte, der Luftfeuchtigkeit etc. abhängig ist). Wenn du alle Parameter berücksichtigen willst, wirst du es nur numerisch lösen können, weil diese Parameter zum Teil wieder voneinander abhängig sind. Nimmst du einige der Parameter als konstant an (z.B. Dichte, Feuchtigkeit) wird es zum einen einfacher, zum anderen ungenauer.

Das einfachste wird sein, du nimmst einen Ballistikrechner und rechnest damit ein wenig rum. Beispiel: 2000 Meter Flug und 2001 Meter Flugweite. Wenn der Höhenunterschied bei diesen Entfernungen z.B. mehr als 100 Meter beträgt, ist die Bahn annähernd senkrecht.

QUOTE
wie weit fliegen die geschosse unter laborbedingungn wenn ich sie waagerecht abschieße,

Auch im Labor gibt es den Knackpunkt Luftreibung. Wenn du den weglässt (Vakuum), fliegt die Kugel beliebig weit, weil sie nicht abgebremst wird.
 
jever
Beitrag 4. Jan 2005, 00:17 | Beitrag #26
+Quote PostProfile CardPM
Oberleutnant
Beiträge: 1.552



Gruppe: Members
Mitglied seit: 31.01.2003


QUOTE(GFF_phoenix @ 03.01.2005, 23:33)
ich kann mir nicht vorstellen, das das so kompliziert sein soll. mata.gif

Es ist noch sehr viel komplizierter.Die Beeinflussung durch den Drall der Kugel haben wir noch garnet mit einberechnet.
Über dieses Thema kannst Du schon fast eine Diplomarbeit schreiben.
Die Aerodynamik ist nunmal keine Schulphysik, und ich habe Dir eine Möglichkeit genannt, wie Du auf realistische Ergebnisse kommst, ohne dass wir hier richtig fies anfangen zu rechnen.
Ich kann Dir eine Formel machen, wenn Du mir cW-Wert gibts und Luftdruck + Temperatur + Luftfeuchte gibts. Aber das kostet Dich einige Dutzend Kästen Bier und Gedult bis zu den Semesterferien im Sommer, denn ich häng da einige Tage/Wochen dran rum.

MfG jever
 
Yoda
Beitrag 4. Jan 2005, 00:20 | Beitrag #27
+Quote PostProfile CardPM
Leutnant
Beiträge: 904



Gruppe: Members
Mitglied seit: 04.03.2003


QUOTE
Aber das kostet Dich einige Dutzend Kästen Bier

Biete hiermit meine Hilfe an  ;)
 
jever
Beitrag 4. Jan 2005, 00:24 | Beitrag #28
+Quote PostProfile CardPM
Oberleutnant
Beiträge: 1.552



Gruppe: Members
Mitglied seit: 31.01.2003


QUOTE(Yoda @ 04.01.2005, 00:20)
QUOTE
Aber das kostet Dich einige Dutzend Kästen Bier

Biete hiermit meine Hilfe an  ;)

OK, angenommen.
Bierkosten damit verdoppelt biggrin.gif

MfG jever
 
Delta
Beitrag 4. Jan 2005, 00:32 | Beitrag #29
+Quote PostProfile CardPM
Elder Forenmen
Beiträge: 7.523



Gruppe: Ehrenmoderator
Mitglied seit: 08.04.2002


Darf ichs auch noch bisschen schwerer machen? biggrin.gif

Da Geschosse in der Regel ueberschallschnell das Rohr verlassen, kann man nicht mal mit ner einheitlichen Formel fuer den Widerstand rechen. Solang das Ding ueberschallschnell fliegt hat es einen nicht unerheblichen Wellenwiderstand zusaetzlich zum Reibungswiderstand. Dann muss man im Unterschallbereich noch gucken, dass man es eiunmal mit inkompressibler und kompressibler Stroemung zu tun hat. Also schon vereinfacht 3 vollkommen unterschiedliche Widerstandsverlaeufe fuer bestimmte Geschwindigkeiten. Noch wilder wirds, wenn man fluegelstabilisierte Geschosse anguckt..... dann haben wirs naemlich noch mit Interferenzwiderstaenden zu tun. Und ganz wild wirds, wenn das Ding aus irgendwelchen Gruenden auch noch Auftrieb erzeugen wuerde.... auftriebsinduzierter Widerstand waere zusaetzlich die Folge.


--------------------
Thou canst not kill that which doth not live. But you can blast it into chunky kibbles.

Limitless are the ways of mankind in its virulent capacities
Ironic it may seem, for us a chance exists to see
In us also lies the capacity to transcend
 
jever
Beitrag 4. Jan 2005, 00:38 | Beitrag #30
+Quote PostProfile CardPM
Oberleutnant
Beiträge: 1.552



Gruppe: Members
Mitglied seit: 31.01.2003


Ich erhöhe die Dutzende um Hunderte biggrin.gif

MfG jever
 
 
 

2 Seiten V   1 2 >
Reply to this topicStart new topic


1 Besucher lesen dieses Thema (Gäste: 1 | Anonyme Besucher: 0)
0 Mitglieder:




Vereinfachte Darstellung Aktuelles Datum: 25. April 2024 - 15:13